Fill This Form To Receive Instant Help

Help in Homework
trustpilot ratings
google ratings


Homework answers / question archive / Instructions Answer one question for each of the readings listed below

Instructions Answer one question for each of the readings listed below

Sociology

Instructions

Answer one question for each of the readings listed below. Responses should be detailed and thorough enough and support your discussion with concrete examples from the readings. Number your responses.

#1 - Purity, Danger, and Redemption

Answer only ONE of the following study questions of James S. Bielo’s article, “Purity, Danger, and Redemption: Notes on Urban Missional Evangelical

undefined

  1. How do James Bielo characterize “emerging evangelicals” as part of a “postmodern” religious movement? How, according to the people Bielo studied, is “postmodernism” different from “modernism”? How specifically do Kevin and Bart seem express their postmodernism? Finally, how in the conclusion of the article does Bielo challenges the distinction between modernism and postmodernism?
  2. How does the “emerging evangelical” movement position itself vis-à-vis contemporary “suburban megachurch” Christianity? What does the emerging evangelical movement suggest to you about the “culture of Christianity” in the 21st century US?
  3. Focus on the “ethnographic” dimension of this article. How did Bielo undertake his “fieldwork” and how does he present his ethnographic description? What do Kevin and Bart teach Bielo about Middletown and Walnut Hills, respectively? How does their lifestyle correspond to their religious worldview?
  4. Come up with your own insightful, probing question on the article and respond to it using concrete examples from the text.

#2 - Calling Everyone to Pray

Answer only ONE of the following study questions on Isaac A. Weiner’s article: “Calling Everyone to Pray: Pluralism, Secularism, and the Adh?n in Hamtramck, Michigan

undefined

  1. Throughout this article, Weiner continually refers to debates over the place of religion in “the public sphere.” What does he mean by “public sphere”? Based on your reading of the article, what role does religion normally play in the public sphere in the US? How does the Muslim call to prayer present symbolic challenges in the public sphere? (Support your discussion with examples from the article).
  2. What are secular (or privatist) arguments that sought to mute the adh?n in Hamtramck? How does the author find these problematic? How, in turn, did the pluralists interpret the adh?n? What did Weiner find problematic in their interpretation? How in turn did local Muslims interpret the adh?n?
  3. How has Hamtramck been transformed by the rise and fall of the auto industry, and immigration in the 20th and now 21st centuries? What does this suggest about the long-term presence of the adh?n in that community? What is Weiner’s underlying argument here? What is your take away from this article

 

JAMES S. BIELO Miami University Purity, danger, and redemption: Notes on urban missional evangelicals ABSTRACT In this article, I examine how urban missional evangelicals in the United States cultivate a sense of place. Being “missional” refers to the desire to be a missionary in one’s own society, an idea that has spread widely through the Emerging Church movement. Proceeding from an ethnographic analysis of two urban pastors, I argue that being an urban missional evangelical means having an intricate, nuanced, but ultimately mediate sense of place. Grounded in a cultural logic that seeks distance from suburban evangelicalism, the urban missional sense of place exists as a lived critique of modernity, which I explore through Mary Douglas’s classic analysis of purity and danger. [U.S. evangelicalism, sense of place, anthropology of Christianity, modernity] “W hat could be truer of placed experience—secure or fragile, pleasurable or repugnant, comforting or unsettling—than the taken-for-granted quality of intense particularity?” Steven Feld and Keith H. Basso conclude the introduction to their landmark collection Senses of Place (1996:11) with this question. At stake is the fundamental difference between physical space and inhabited place. As an analytic, “sense of place” promises anthropologists that, through careful ethnography, they can trace, unpack, and convey “local theories of dwelling ... ways of fusing setting to situation, locality to life-world” (Feld and Basso 1996:8). Or, as Basso says elsewhere, “Dwelling [consists] in the multiple, ‘lived relationships’ that people maintain with places, for it is solely by virtue of these relationships that space acquires meaning” (1996:54). In this article, I bring that promise to an ethnographic site Feld and Basso may not have imagined: the urban frontier of U.S. evangelicalism. I focus on ethnographic portraits of two urban pastors, Kevin and Bart. Both men organize their efforts around the idea of being a missionary to their own society—“being missional,” as they would have it. Their missional impulse is tied closely to a movement of religious and cultural critique that arose among U.S. evangelicals in the late 1990s, the “Emerging Church” movement (Bielo 2009a, in press). I begin with a description of this movement, setting the context for two neighborhood tours Kevin and Bart led me on during the summer of 2009. Through an ethnographic recounting of these tours, integrated with the cultural–religious biographies of my two guides, I pursue an integrated argument. Being a missional Christian entails having an intricate, nuanced, but ultimately mediate sense of place. That is, the model of dwelling these Christians cultivate, although rich in detail, is not an end in itself but a means to the well-rehearsed evangelical end of reaching “the lost” (Harding 1987). This sense of place serves as a lived critique of two dominant and related cultural configurations: conservative evangelicalism and the suburbanization of white, middle-class U.S. society. As a cultural logic, this critique extends to the conditions AMERICAN ETHNOLOGIST, Vol. 38, No. 2, pp. 267–280, ISSN 0094-0496, online C 2011 by the American Anthropological Association. All rights reserved. ISSN 1548-1425. Q DOI: 10.1111/j.1548-1425.2011.01305.x American Ethnologist • Volume 38 Number 2 May 2011 of modernity. To unpack it, I look to the comparative work occurring among anthropologists of Christianity and to a verse from anthropology’s scriptural canon: Mary Douglas’s Purity and Danger (1966). The priority assigned to place by Emerging evangelicals should also be read as a chapter in the ongoing story of U.S. Protestant interventions in civic society. In Weberian terms, evangelicals have always existed as “innerworldly ascetics”: seeking a level of deep spiritual commitment while still participating in the institutions of modern life (Pals 2006:173). Or, to use a reliable evangelical tagline, they seek to “be in the world but not of it.” But how does this inner-worldly posture play out in lived sociological contexts? Omri Elisha (2008b) and Jon Bialecki (2009) have offered ethnographically informed versions of how and why evangelicals become “socially engaged” (Elisha 2008b:155). I reference the details of Elisha’s case because the cultural response he describes provides an illustrative foil for individuals, like Kevin and Bart, and for communities that have internalized the cultural critique that circulates through the Emerging Church movement. Elisha focuses on the faith-based activism of white, middle-class, suburban, conservative evangelicals in Knoxville, Tennessee. He argues that the familiar condition of “compassion fatigue” gets refigured among them as “the gaps between one’s moral ambitions and the conditions of existence that reinforce and simultaneously threaten to undermine them at every turn” (Elisha 2008b:155). They manage this fatigue, self-consciously but uneasily, through “the twin imperatives of compassion and accountability” (Elisha 2008b:156). They direct their outreach activities primarily toward economically distressed inner-city residents, creating a situation in which they, as “suburban churchgoers[,] interact with cultural strangers affected by conditions of poverty, distress, and marginalization” (Elisha 2008b:156). To capture their dilemma, Elisha depicts the following ethnographic scene: the first-ever meeting of the Samaritans of Knoxville, a faith-based activist organization founded by a former pastor. At one point during the meeting, a man named Howard stands up, the only African American in a room of 18 local church leaders. Howard proceeds to criticize what he has been listening to: “I’m the one who has to clean up the blood of the messes you make when your churches just drop in, drop off, and leave the community behind” (Elisha 2008b:161). The white suburban evangelicals have little of comfort to offer Howard in response, and the meeting continues with his critique dismissed. The ethnographic case of socially engaged evangelicals that I present here contrasts with the version captured by Elisha and suggests the need for an alternative analytic, one that includes attention to the discourse of being missional and the sense of place that it travels with. 268 Emerging evangelicals Kevin and Bart both see themselves as urban missionaries. This identity is central to their sense of self and works to organize their everyday religious subjectivity. Theologically, Kevin and Bart are far from twins. Both are evangelicals, if by that one means they experienced a born-again conversion; their understanding of history, morality, and eternity are channeled through Jesus; and they understand the Bible as God’s most direct and complete revelation to humanity. However, they interpret their Christ-centered faith quite differently. Kevin is committed to a Calvinistic Reformed theology that stresses human depravity and the doctrinal exclusivity of the event of Jesus’s substitutionary atonement for eternal salvation. Bart is far more ecumenical. He identifies as a “post-Christian,” affirming love and grace as the definitive attributes of God, and questions doctrinally centered soteriologies: “You can be a Christian and believe all the wrong things.” I describe both men more fully below, but I highlight their theological divergence here to recognize that the Emerging Church movement has influenced a wide range of evangelical identities. It is through this movement that Kevin and Bart have refined their understanding of being missional.1 The “Emerging Church” appeared on the U.S. religious scene in the late 1990s as a gathering point for Christians convinced of a decided shift in U.S. society from modernity to postmodernity. Modern and postmodern are widely used by Emerging Evangelicals and are typically understood in epistemological terms: the former equated with Enlightenment philosophies of scientific rationalism and the latter with an era defined by collective doubt about the human ability to know absolute truth with absolute certainty. In 1995, Leadership Network, a popular evangelical institution among local church leaders, launched a project called Young Leaders Network (YLN). YLN consisted of pastors, youth ministers, church planters, and church consultants— primarily white, male, well educated, urban, and under 40. It saw itself as addressing a single, detrimental problem facing the future of U.S. Christianity: a gaping dissonance between the youngest generations in the country and the organization, style, and interests of conservative evangelicalism. In short, they argued that Christianity as it had been imagined and practiced in the United States no longer worked. The YLN conversations quickly spread via conferences, informal networks, and Internet communities to other Christians dissatisfied with their church cultures. The critique at play quickly exceeded the trope of a generation gap and gathered under its umbrella scriptural hermeneutics, theological method, liturgical practice, witnessing style, and ecclesiological structure. All were cast as part of the same self-examining moment among “younger Purity, danger, and redemption evangelicals” (Webber 2002). Divisions within these ranks soon appeared. Some thought the theological questioning unnecessary and sometimes heretical. Some were interested in one aspect of the cultural critique but not other elements. Others saw the effort as part of an unavoidable shift from a modern to a postmodern Christian consciousness. As the 21st century got underway, the first intentionally Emerging church communities formed in a diverse collection of U.S. cities: Santa Cruz; San Diego; Minneapolis; Kansas City; Houston; Seattle; Grand Rapids; Washington, DC; Philadelphia; Raleigh; and Cincinnati. In 2002, the first international Emerging Church conference was held in San Diego in conjunction with the National Pastors Conference (one of evangelicalism’s most widely attended annual meetings). Christian and secular media consistently reported on the movement by 2002, with attention surging in late 2005. And the world of Christian publishing witnessed the same slow appearance and sharp increase in interest, evident in contracts for ongoing Emerging Church book series with three of the largest Christian publishers in the United States. Globally, the movement is undeniably transnational in the Anglophone world, and several influential Emerging Church authors and practitioners are from Canada, the British Isles, Australia, and New Zealand.2 Ultimately, I find it no exaggeration to say that the Emerging Church has become the most vocal, influential, and debated movement among U.S. Christians since the Religious Right’s rise to political and cultural prominence in the late 1970s (Harding 2000). Unlike other recent evangelical inventions—for example, being “Purpose-Driven” (Balmer 2006) or “Seeker-Sensitive” (Sargeant 2000)—the Emerging Church is a thoroughgoing cultural critique of Christianity as imagined and practiced by the traditions (mainline, evangelical, and charismatic) that controlled the order of U.S. Christian discourse in the 20th century. Although Emerging Church ideas and practices have been consumed by a diverse body of evangelicals, an intensely common ground exists around the idea of being a missionary in one’s own society. Emerging evangelicals most often trace the origins of the “missional” concept to Lesslie Newbigin (1909–98), a British Anglican priest who spent most of his life as a missionary in India. In the early 1980s, he began speaking and writing on the challenges of evangelizing in the West, which he considered just as difficult as witnessing to non-Westerners (if not more so). Newbigin attributed this difficulty to the epistemological suspicions about religion produced by the Enlightenment’s colonizing scientific worldview. He concluded that all Christians are missionaries and that successful mission work in the West meant learning the language and culture of the local mission field, wherever that might be and however familiar it might seem. Newbigin first articulated this missiology in two treatises: The Other Side of 1984 (1983) and Foolishness to the Greeks (1986), the latter the more in- • American Ethnologist fluential in the United States (Goheen 2000). He describes “modern Western culture” against a Weberian grain: “The result is not, as we once imagined, a secular society. It is a pagan society, and its paganism, having been born out of the rejection of Christianity, is far more resistant to the gospel than the pre-Christian paganism with which crosscultural missions have been familiar” (Newbigin 1986:20). The organizing logic is that Christianity is no longer the default worldview among Westerners. Quite the opposite: It is widely considered to be anachronistic, irrelevant, destructive, or worse. Emerging evangelicals have treated Newbigin’s missiology as a methodological critique of conservative Christian evangelism. They decry a wide range of common witnessing practices: street preaching, handing out Bible tracts, delivering finely tuned conversion speeches, using hyperlogical apologetics, and using weekly congregational events as the entre?e to church. For those self-consciously striving to be missional, these methods suffer from several problems: a failure to understand the shift in U.S. public consciousness from modernity to postmodernity; an inability to effectively use the mediums and idioms attractive to a postmodern audience; and the lack of meaningful, lasting personal commitments. Being missional, they assert, means seriously cultivating relationships—not before or after conversion attempts but in place of them. To accomplish this goal, they advocate mimicking the acculturating foreign missionary: settling into a locale and becoming intimately familiar with a place, its people, and their language. The issue I take up here is how this orienting identity of being missional produces a distinct sense of place among urban evangelicals, one that pulls together a consciousness of race, class, economy, history, and a host of other social dynamics. I begin with the stories of two pastors, whose everyday religious subjectivity involves them in managing their relation to place. Two pastors “A catalyst for a hurting city” Kevin is extremely affable. Actually, it is not so much geniality or sociability that makes him so disarming but gentleness. He projects this mildness despite a tall, brawny frame and a full brownish red beard that conjures Viking images. Perhaps his voice and speech temper his physicality: soft, measured, reflective, articulate, but not overly intellectualized. I first interviewed Kevin in January 2009; he was 28 and days away from being a first-time father. Kevin grew up “nominally Christian” in a mainline Presbyterian church and experienced a born-again conversion through the neocharismatic Vineyard Fellowship in the mid-1990s. In his vocation as a pastor, 2005 was a turning point for Kevin. Two years earlier, he had graduated from 269 American Ethnologist • Volume 38 Number 2 May 2011 college with a history degree and had been working as a regional director for the Young Life organization. As the director, he trained roughly thirty volunteers a year and organized Young Life ministries for five high schools in the Dayton–Cincinnati, Ohio, corridor. Working for Young Life was satisfying but not the same as being a pastor for his own congregation, an aspiration he had had since the age of 16. This desire made the offer of a senior pastor position with a suburban church in Florida all the more tempting. It was the kind of offer he had longed for. But he declined. His deciding impression was that the congregation—whose worship space was located next to an affluent, residential subdivision—was composed of wealthy, white individuals accustomed to paying for anything they wanted. He considered them beneficiaries of comparatively enormous economic privilege and did not want to be complicit in extending this advantage to their religious life. In May 2006, with the support of his wife and three other married couples, Kevin decided it was time to start his own church. For several months, this group prayed for “direction” and, then, in August, started weekly Bible study in Kevin’s brother’s home with six other married couples. Participation quickly grew to 30 people, and Kevin was convinced his decision was the right one. It was then that Kevin experienced a “second conversion” about his “primary motivation in life.” Prior to this point, if asked about his motivation, Kevin would have given a predictable evangelical response: “To convert unbelievers.” Now, his response is, “To glorify God.” His reorientation stemmed primarily from his exposure to an evangelical church planting organization, Acts 29. In late 2006, Kevin applied for inclusion in the organization’s network. Acts 29 was founded in 2000 as a resource for those desiring to “start churches that plant churches.” As of October 2010, the network included 210 churches in 42 states. Although it is interdenominational, Acts 29 is not particularly ecumenical, requiring members to affirm a strict theological commitment to Reformed Calvinism. The network’s “About” web page provides a 22-point set of belief statements along with an 18-point set of statements on “what ... Acts 29 churches [do] not believe.” These lists are preceded by a pithy self-identification: “We are first Christians, second Evangelicals, third Missional, and fourth Reformed.” The organization’s summation of being “Missional” echoes Newbigin’s concern with the particularities of specific cultural contexts: We believe that our local churches must be faithful to the content of unchanging Biblical doctrine (Jude 3). We believe that our local churches must be faithful to the continually changing context of the culture(s) in which they minister (1 Corinthians 9:19–23). We believe that our mission is to bring people into church so that they can be trained to go out into 270 their culture as effective missionaries. [Acts 29 Network 2010] Applying to join Acts 29 is nothing if not elaborate. In addition to Kevin, I interviewed seven other Acts 29 church planters, all of whom emphasized the “intensity” of the assessment process. Along with a rigorous theological interview conducted by existing Acts 29 planters, the hopeful pastor and his wife are interviewed together and then separately about their marriage and family life. In January 2007, Kevin’s church plant, the Oaks Community Church, was approved, and in September 2007 began holding weekly worship services. Its name—the Oaks—is a reference to Isaiah 61:3 and Kevin understands it as a way to publicly signify church members’ concern with place: “They will be called oaks of righteousness, a planting of the Lord for the display of his splendor” (New International Version). As of September 2010, the Oaks had attracted roughly three hundred people for Sunday morning worship and maintained 12 small groups, each with 15–20 participants, that met in members’ homes. Middletown, Ohio, is the home of the Oaks. This city will probably never be a tourist mecca. Aside from the absence of any natural wonders or historical landmarks, it is iconic of a failing industrial United States, that is, if Forbes magazine is correct. In December 2008, Forbes published a special real-estate issue on “America’s Fastest-Dying Towns: Ten Spots Where Jobs Are Vanishing, Incomes Are Dropping and Poverty Levels Are Rising” (Woolsey 2008). According to the U.S. Census Bureau’s 2005–07 American Community Survey, Middletown ranks economically with the lowest of the low. A city of 50,000 people, it epitomizes economic decline. Among other disparaging statistics, Middletown has seen “the seven-year poverty rate jump from 12% to 22%”; only “12% of residents possess a bachelor’s degree or better”; and “the town’s median household income is $37,000.” Much of Middletown’s distress—in both statistical and popular imaginations—is due to declining production by AK Steel, a gargantuan steel factory that snakes along the city’s southern and eastern edges. Though Kevin is a southwestern Ohio native, Middletown was not chosen for him. He chose it. Not far away are far more inviting, prosperous locales. Of all places, why plant the Oaks here? After all, it might be hard filling the coffers in a place like this. Middletown is one of ten communities within a tenmile radius that the Oaks hopes to draw people from. It is located on the Interstate-75 (I-75) corridor, roughly thirty miles south of downtown Dayton (also one of Forbes’s unlucky ten) and about thirty miles north of downtown Cincinnati. Kevin’s official church planting proposal—the document that accompanied his acceptance into Acts 29 and that is used to solicit financial sponsorship— emphasizes the struggling quality of Middletown but also its strategic location. The proposal includes a map of the Purity, danger, and redemption region, below which—in bold, italicized letters—are the words of the county’s executive director of transportation: “What we are doing is really thinking about what the whole I-75 corridor between the two beltways looks like and its impact on job growth in southwestern Ohio. Long range, that corridor—that handful of exits—will be the center of the universe.” Next to these official, authorizing words are Kevin’s, describing the logic behind planting here: Middletown is a very diverse region in many ways, and it currently sits under a cloud of depression—both economically and socially. While surrounding cities are seeing steady growth, Middletown has seen little positive growth or development. The vast financial and social needs of the city provide very real opportunities to serve the area, and to be a catalyst of growth, development, and change for a hurting city. If we can help enact a healthy change within the city of Middletown, we will most certainly have a positive impact on the surrounding communities. The missional attraction to Middletown begins by defining “needs” socioeconomically, playing off the Christian metaphor of being a beacon of hope and light. Effecting change in Middletown is promised to be a witness for nearby communities. The most significant observation about those communities—and the thread that connects Kevin’s ongoing efforts with his 2005 decision to forgo Florida—is their status as upwardly mobile, suburban, insular locales. Most of the remaining nine communities in the proposal’s geographic radius emulate and approximate what Setha Low (2003) described as “fortress America,” a collage of gated and walled new-growth residential developments that fuel class and race segregation. Kevin’s decision to settle in Middletown was very much about bypassing the places that, in the conservative–evangelical– megachurch imagination, are more likely candidates for starting a new church. “All I needed was dinner” Bart is a captivating speaker. It is hard for a listener not to hang on his words, get lost in story, move with steadiness to chair’s edge. For an ethnographer, listening to him frankly borders on distracting. You sometimes wish he were slightly less engaging, allowing you to maintain your familiar routine of gliding between descriptive field notes and analytical notations. A portion of his income comes from speaking at churches and public forums across the country. Imagine the energy of an enthralling stage presence, bottled and placed in a small room where you are the only audience. He is a storehouse of one-liners, not cliche?d or tired but inventive and tied tightly to the details of your shared talk. Not overly tall or muscular, he is both. Now 45, he continues to move with the grace of a natural athlete. • American Ethnologist Bart grew up near Philadelphia. His father was (and is) a well-known evangelical author and speaker, a kind of “larger than life figure,” but he was not Bart’s avenue into born-again Christianity. Not hugely popular in early high school, Bart earned hallway fame by leading the school’s soccer team. But he remained “suspicious” of this social success, remembering the worst of how adolescents could treat one another. The only classmates he saw living counter to the tenuousness of fame and isolation were the evangelical students. He “fell in with this Christian crowd” and “loved the way they were in community together.” It was “the only place where the strong took care of the weak.” A year later, he had his own born-again experience. His conversion was not, however, the most significant moment of his religious youth. The summer before his senior year of high school, he participated in a summer camp ministry in Camden, New Jersey. Living and working in the “straight up ghetto,” his first “real encounter” with poverty was a “life-changing revelation.” The reality of poor living conditions was never broached among his friends and mentors at church, so he expected to return home the bearer of new, eye-opening news. He found, though, that his peers and the adults in his community were not unaware of such poverty. At best, they felt helpless to address it, and at worst, they were apathetic about it. Morally, he was “disillusioned” with what he perceived to be suburbia’s utter dismissal and lack of care. Theologically, the “Calvinist” emphasis on divine sovereignty he had been socialized into did not square with his Camden experience. At 18, Bart became convinced that his life’s vocation was to live and work with poor people. He spent his college summers doing mission work in inner-city Philadelphia. After college, he moved to Minneapolis to work for an urban youth ministry. He spent three years there before moving back to Philadelphia with his new wife to start an urban ministry aimed at helping congregations develop better youth programs. Despite the organization’s success, Bart had a dawning realization that his impact was primarily, if not solely, on the people doing ministry, not those being ministered to. In 1997, Bart founded a more ambitious program. It placed teams of five or six young adults in oneyear residential internships with inner-city churches. Their job was not to “start anything” but simply to volunteer in community agencies, become part of these churches, and intentionally form ongoing relationships with poor people. Bart oversaw this program in five major cities for ten years. Three things became increasingly apparent to him as his tenure with the organization matured. First, although he was deeply connected to dozens of inner-city churches and communities across the country, his travel schedule and his focused energies kept him from genuinely knowing any of his own neighbors. Second, he increasingly saw his role as “subversive” because most of his volunteers were “hard-core evangelicals” who had expected their 271 American Ethnologist • Volume 38 Number 2 May 2011 responsibilities to center on proselytizing but quickly (and frustratingly) learned that “loving people” was the goal. Third, he was no longer “living the life of an inner-city missionary” and, even more discouraging, “hadn’t really known a poor person for 15 years.” The result was a retreat to “sadness” and the decision to step down as program director. After doing so, he and his wife—as well as their 13-year-old daughter and 10-year-old son—“moved to a rich Philly suburb” where he “felt like a stranger.” Two years later, in 2005, they decided “as a family” that they needed to recommit to the “original idea” of “living with poor people and loving [their] neighbors.” In the summer of 2005, Bart performed a wedding in Cincinnati. At the reception, two old friends asked what he was doing, and Bart explained his family’s dilemma. The couple invited them to move to Cincinnati and pursue the “original idea” there. The couple lived in West Walnut Hills, an economically depressed, primarily African American neighborhood near the city’s downtown.3 The place fit the conditions of the “original idea,” and the family’s decision was easy: Move to West Walnut Hills. Bart insisted on not starting a church. He wanted to do something that local residents “needed” and that would “draw in community.” His first idea was to open a thrift shop, something he had seen done in other cities with success. Six months of planning produced only the unfortunate recognition that a thrift shop was not economically viable. His second idea, suggested by a local resident, was to open a laundromat. After three months of planning, the same recognition emerged. Frustrated, he and his wife decided to host a community meal, with the hope of “drawing people together.” Bart invited 20 of the local homeless and unemployed people he knew from nine months of living in the neighborhood as well as ten friends who were interested in “being in relationship” with this collection of people. In his words, “It was a disaster.” Local residents were encumbered by distrust and suspicion toward each other, their hosts, and the other guests. The other guests were at a loss for what to say to residents or how to say it. Undeterred, Bart hosted another dinner. And yet another. The Walnut Hills Fellowship (WHF) now meets every Monday for dinner. “It’s the most motley crew you ever want to see.” Bart explained, “I was so preoccupied looking for something to start, it turns out all I needed was dinner.” As Monday night dinner became a regularity, Bart sought further ways for WHF members to support each other. For example, he started organizing housing for community members who need reliable residence. His group has remodeled four houses within a four-block radius. Each had been condemned by the city and cost about $10,000 to purchase. Forty men and women, a collection Bart describes as “terminally broken,” show up each week for the Monday dinners. “These people can’t be fixed. Their life is already decided. But nobody should die without a friend. 272 That is what we are, professional lovers. I’m like the father of one, big dysfunctional family.” He recalled a woman in their community as a case in point: 52 years old, living with a disability and failing health, uneducated, formerly imprisoned for multiple felonies, no home, no car. “Do you think anyone is going to hire her? Come on! Cutting back to getting drunk once a week and sleeping inside on a real bed, that is a success story here.” Referencing her shift from homelessness to a $25-a-month Housing and Urban Development apartment organized and furnished by the WHF, Bart continues, “She’ll never be self-sufficient, but at least she has some dignity now.” She is living on her own, in her own place, for the first time in a long time. ∗∗∗ Kevin’s and Bart’s stories are not identical. As I note above in introducing the two pastors, their theological renderings of Christianity collide as much as, if not more than, they converge. Kevin settled in a primarily working-poor white city and Bart in a predominantly un(der)employed black borough in a larger city. Kevin intended from the start to plant a new church. Bart wanted as few institutional trappings as possible. Kevin is a paid clergyman, and Bart is not. In some ways, though, their stories overlap. Both are white, middle-class, family men who voluntarily moved into settings unlike those in which they grew up and where secular logics of upward mobility aspire. Both seem to embody a grassroots approach to idea(l)s of organizing, community, and change. However, I want to emphasize another similarity. Kevin and Bart both narrate a second conversion. Kevin’s entails a shift from “converting unbelievers [to] glorifying God.” Bart’s entails a reorientation of consciousness about privilege and despair. For both men, this change produced an antagonism toward the evangelical subculture in which their religious lives began and matured. Observe, for example, that both men are evangelistic, a hallmark of born-again Christianity, but neither proselytizes in the received manner of conservative evangelicalism. The tension they maintain with the evangelical subculture has prompted both men to start new ministries (albeit structurally distinctive ones). Finally—and this is crucial for the larger argument I am making—the tipping point for both men is grounded in a dissonance with U.S. suburbia, itself an index of the conservative evangelicalism they want distance from. (This is a tipping point as both a narrative device of self-presentation and an orientation for ongoing practice.) Their moves into Middletown and West Walnut Hills emerge from a self-conscious desire to escape suburbia. This desire and the cultural critique of evangelicalism that it trafficks through are actualized in the sense of place Kevin and Bart cultivate as urban missionaries. Purity, danger, and redemption Two places “My job is not to fix Middletown” On June 25, Kevin led me on a three-hour, winding, but ultimately circular driving tour of Middletown. The weather was staggering: 90-plus degrees, high humidity, little cloud cover, no breeze. It was perfect for swimming. The previous week, along with 13 other pastors and community leaders, Kevin had attended three three-hour workshops on “racial reconciliation” hosted by the city council. Details of those workshops were fresh in his mind and clearly occupied his attention, as he frequently connected them to our roaming conversation. Kevin already knew the racial demographics of Middletown (about 10 percent African American, a very small constituency of Hispanics, and the rest white); in fact, he had gathered these data as a matter of course when planting the Oaks. Much of the council’s discussion had turned “political,” which Kevin kept a distance from. He is not, he continually found occasion to tell me, very interested in the political commitments conservative evangelicals typically foster.4 The main lesson Kevin took from the workshops was the idea of “white privilege.” This, he felt, was what he and his congregation most needed to hear about. Toward the end of our drive, Kevin turned onto Minnesota Street (“the Sota”). “This is the worst street in Middletown,” he noted, referencing various forms of known and undiscovered criminality. It also runs directly through the largest African American neighborhood in the city. Kevin turned the car around at Douglass Park: a modest, partly grassy affair with a jungle gym and a dozen or so water streams that shot up from the ground. It was busy but not diverse. Everyone milling around, from adults carrying picnic baskets to children splashing and laughing in the water, was black. We paused for a moment, and Kevin reminded me that 15 minutes earlier we had passed Sunset Park—a much larger complex, quite verdant, with a large community pool (large enough to require two lifeguards), and nearly all white patrons. He explained that the city used to fund two community pools, until a massive state budget cut forced one to close several years earlier. The recompense for Douglass Park was the series of water spouts sporadically shooting through half-dollar-sized holes drilled in the concrete. He offered this as case-and-point for the council’s focus on privilege and an example of precisely what his congregants should pay attention to. Soon after we left, proceeding down the Sota, Kevin pointed to a large football stadium on our left. The hosts of the racial reconciliation meeting claimed that high school football games are the only consistent occasions when whites and blacks occupy the same social space in Middletown. Kevin, who tries to attend every home game as part of his missional work, affirmed this and went on to say that seating is not segregated. “Everybody’s mixed up together. It’s great.” • American Ethnologist Central Avenue—downtown’s main drag—is two miles from the Sota. Since December 2007, a two-story unit on this strip has been the rented home of the Oaks’s weekly worship services and church events. As you ascend the wide, steep, wooden stairwell to the second-floor worship space, a series of black-and-white photos line the wall to your right. They are artfully composed. The first shows a sign reading “Middletown,” one of several marking various entry points into the city’s corporate limits. The final picture, aligned with the last step, captures the high school football stadium, lit brightly for a night game. The space is not particularly ideal for the Oaks. It is surrounded mostly by empty storefronts and struggling retailers, not a generous scattering of dining options for Sunday, lunch-hungry churchgoers. It has several size-related problems. The stage for the worship band is cramped. The acoustics are mediocre at best. There is no open, communal area for people to congregate in after worship services. The back rooms for nursery and Sunday school are too small and crammed close together. The temperature is not centrally controlled. In the summer, two large vents in the worship area pump in plenty of cool air, accompanied by a distractingly loud HUMMM. In winter, a single small heat vent near the ceiling must be opened on Saturday night to make the space “bearable” by Sunday morning. Still, Kevin wants to stay. Apart from the very reasonable monthly rent, he emphasized values of stability (“Every other church has left downtown. We don’t want to do that”) and distinction (“We have 300 people down here every Sunday. When else does that happen in Middletown? It doesn’t”). Earlier in the tour, when we first pulled onto Central Avenue, Kevin described the urban landscape along the strip: “like a bomb went off downtown, the further away you get, the nicer things get.” This became evident on our way to eat lunch, as we drove farther down Central, past the worship site. Kevin wanted me to eat at his favorite restaurant, a locally owned Italian-themed place. “When you hear people talk about being missional,” he waxed reflectively, “one thing you’ll hear is finding places to be a regular.” Kevin eats at the restaurant about eight times a month, brings out-of-towners (visiting pastors, friends, family, and hungry ethnographers) there, and has it cater church events. He recalled with no small amount of fondness the first day he was greeted there by name. The owner—who spoke familiarly and in detail with Kevin during our visit—has worked hard to improve the building’s exterior, one reason Kevin likes to support the business. He has, however, been unable to purchase an adjacent, clearly bedraggled house. Kevin described it as a “crack house or something, lots of drug deals moving through,” using the ostensible illicit epicenter as an example of a pervasive problem in Middletown— drug abuse. “Marijuana is huge here. Even middle-class folks are pretty big pot smokers; it’s just part of the culture.” 273 American Ethnologist • Volume 38 Number 2 May 2011 Lunch cost $20 and consisted of two Italian sandwiches and two drinks. We ate inside, escaping from the midday heat. Most of our talk focused on how Kevin’s coming to know “the culture” has affected what he does as a pastor. He mentioned two examples that especially excited him. First, Kevin has become much more self-conscious about his sermon writing. Acts 29 pastors, and others tied to the revival of Reformed Calvinism in America (Hansen 2008), are well known in the Emerging Church movement as expository preachers. They tend to explain scriptural texts week to week with incredible deliberateness (critics might say “tediousness”). Kevin once spent five months teaching 1 Peter, a New Testament epistle of only five chapters. On an average Sunday, he aims for 40 minutes of preaching, which distills 15 to 20 hours of weekly preparation. But the longer he preaches in Middletown, the more he worries this style is “too linear, too academic.” Tim Keller is a church planter in Manhattan and a hero to many Acts 29 pastors. However, Keller’s approach has proven to be an inexact model for Kevin’s preaching. Keller’s audience does well with New Yorker and Economist references. Time, Kevin suspects, would be a stretch in Middletown. He also haggles over words in a new way. “Things like ‘glory’ and ‘gospel’ have all this meaning for Christians that other people are missing.” His solution has been to recruit a proofreader who is not stingy with corrections—his wife. She diligently scans each sermon for words, phrases, and references that might exhaust the knowledge and patience of poorly educated Middletowners. Kevin’s second example frets about class and race more than education. When he first planted the Oaks, Kevin drove a baby blue Honda Accord. The longer he drove in Middletown, the more he worried about looking like “too much of a yuppie.” After consulting with his wife, he made a change: to a black Chevy pickup truck. Our mode of transport that afternoon, it is no small-bed, comfort-first, low-sitting, S-10 model. It is a bulky, metal, step-up-to-get-in, grumbling-engine, move-it-or-lose-it kind of truck. Kevin’s anxiety has shifted recently to appearing “too redneck” to African Americans. This is precisely the kind of missional dilemma he presents to his congregants. “How would we feel if we pulled up and the church parking lot was filled with BMWs? If you were going to be a missionary in Africa you’d pay attention to these things.” When we left the restaurant in the black Chevy, Kevin drove to a house he and his wife wanted to purchase. He and his family live six miles from the worship space on Central, outside the Middletown city limits. He is anxious to move but cannot afford to buy a new home until the family’s current one sells. Most pastors in the city, he reported with a tone of deep regret, commute nearly 20 miles from West Chester or Mason: largely affluent exburbs of Cincinnati and places Kevin intentionally avoided when planting the Oaks. The house of interest was located farther down 274 Central, away from where the bomb detonated, in an area where “average, everyday, working people live.” The neighborhood did, indeed, have an averageness to it. It was no Sota—less than ten minutes away—with its rusty, broken fences and crooked, crumbling gutters. But there was nothing flashy or extravagant about it either. The house Kevin and his wife hoped to buy was a two-story brick home with a small veranda upstairs. The small front yard boasted a large, canopied oak. The back yard was much bigger and included an ample kitchen garden and covered back porch. The asking price was $130,000, a considerable bargain so long as things like electric and plumbing were in good order. Kevin used the house as an opportunity to explain the Middletown housing market. A steep economic decline tied to AK Steel’s downsizing of labor and production had sparked a sizable emigration and a large dip in housing costs. One result has been an influx of young, first-home-buying families. This is good news for the Oaks. Kevin consistently encourages congregants to buy homes in the city, and, as of May 2010, about half of the Oaks’s members had done so. Throughout our drive, he pointed out houses that church staff and lay members had recently moved into. As we toured Middletown, Kevin made two comments that seem appropriate bookends for his sense of place. Driving past the AK Steel plant, on our way to the Sota, Kevin said after a moment’s silence, “To be a church planter, you have to be an economist, you know? You have to be a sociologist and a demographer and an urban planner.” I asked in response, “Did you get more than you bargained for?” He replied, “Absolutely. I wasn’t really prepared for this.” Indeed, Kevin’s intimacy with Middletown has formed through a process of learning local social, economic, and historical details. Not only did he seamlessly rehearse housing fluctuations, racial residential patterns, and employment trends but he also touched on voting patterns, homelessness, urban–suburban trends, and union culture. As the tour ended and we headed back to where my car was parked, Kevin explained with focused intensity, “My job is not to fix Middletown. As much as I would like that, that’s not what I’m here for. My job is to find out what these people’s deepest fears are. And, my job is to find out what their greatest hopes are. And then I need to show them why Christ is more than either one of those. That’s my job.” Kevin’s sense of place is mediated by a clear idea of purpose. All his observations and understandings pass through the filter of his “primary motivation ... to glorify God.” “If you are going to care for people, you have to care about what they do” On August 7, Bart led me on a two-hour, winding, but ultimately circular tour of West Walnut Hills. The weather was pleasant, nothing like the oppressive heat six weeks earlier in Middletown. I arrived at Bart’s office around noon. He Purity, danger, and redemption was finishing his prior appointment—an amateur journalist had flown from Colorado to interview him. When we left his office a few minutes later, Bart turned to me: “We’re just gonna take a walk. Okay?” We had gone just a few hundred feet when Bart paused on the sidewalk next to a one-way, three-lane road. His finger in the air, he traced the boundaries of the neighborhood—major street arteries going east and west, freeways going north and south. The streets, houses, businesses, services, people, and lives in this area set the everyday context for the WHF. After walking for about five minutes, we turned north on a quarter-mile-long residential street, where Bart lives with his wife and teenage children. Their house is half of a thin, tall, two-story brick duplex. The couple that first invited them to Cincinnati live in the other half. The house directly north of theirs belongs to a WHF member, as do two others on the street. Numerous houses are abandoned, their emptiness signaled by grossly overgrown grass, shrubs, and trees. An older couple who have been in the neighborhood most of their lives live toward the end of the street. Bart and his wife know them well and have invited them to Monday dinner several times. But the couple remember when the neighborhood was a prosperous place and want nothing to do with its current state of disarray. Bart, in what would become a familiar conversational routine, had a ready explanation: “When you get involved in [residents’] lives, their chaos gets all over you. That’s why middle-class black families are so hard to get involved.” The suggestion is that racial obstacles are hard enough to overcome in a segregated city without inviting others’ burdens. We turned the corner, and Bart immediately pointed to a nearby apartment complex. He called it “the worst building in the whole neighborhood,” by which he meant it was the scene of crack-cocaine dealing, violence, and the species of traveling mayhem that trails such activity. It was there, two years earlier, that two ministry interns working for WHF got caught in the middle of gunfire on their way home one night. Luckily, the souvenir bullet holes were restricted to car doors. We turned south, heading back in the direction of the office. After a hundred yards or so, the first instance occurred of what would become a repeated chorus—Bart encountered a familiar face. This time, it was an older black man in a motorized wheel chair, accompanied by a small dog. They greeted each other warmly and shared a brief informational exchange. While walking away, Bart explained that the man rents out several local houses and is “a good guy.” He had barely finished speaking when we paused again. Bart pointed to a sardine line of six two-story townhouses, all built within the last three years. Most were painted a floral tone that sharply juxtaposed the nondescript beige, white, and gray surroundings. They seemed an untainted sore thumb. All but one remained unoccupied. Bart, again, rescued me from confusion. The city had • American Ethnologist used a government subsidy to build them, then placed economic restrictions on residency—one could not make too much or too little money to qualify. The asking price had already dropped $20,000, and Bart suspected the decline would continue, as well as the units’ vacancy. He reckoned the whole thing a parade of governmental stumbling. A few hundred yards farther, we reached the main artery that travels east. On every visit I have made to West Walnut Hills, irrespective of the time of day or day of the week, a congregation of young black men has lined this street one block west: milling about, talking, and rarely laughing. Before I could pause and ask, Bart said knowingly, “We need to keep moving.” (Later, he would add to his verbal mapping of this corner, “I don’t walk that direction with people I don’t know ... the place is drug central, anything you could want.”) We continued for a half-mile or so, encountering a series of businesses and a flurry of sidewalk socializing. Everyone was black, but we saw multiple generations and equal numbers of males and females. The businesses, Bart was quick to say, “are not real businesses,” pointing out wig shops, a dollar store, and a check-cashing service. I saw a CVS drug store one block east and, intending a bit of irony on the ubiquity of chain retailers, quipped, “I see CVS found their way in.” Before I could attempt a laugh, Bart responded, “Thank God for it,” drawing my attention to the economic and consumer stability offered by a corporate entity. He gestured toward a Kroger grocery store a block farther east, assigning it the same significance. He explained how the Cincinnati-based Kroger Corporation tries every year to shut this store down, but community response keeps it open. (At lunch, Bart would compare the abundance of the store’s alcohol aisle to its minuscule vegetable section.) The sidewalk bordering the Kroger parking lot was the busiest scene of the entire tour. A small corner of the lot hosted a man selling jewelry from a wooden stand. Bart greeted a middle-aged black man and exchanged a few quick words about finding a place to live. Mixing regret and hope, Bart informed the man that one-bedroom apartments are hard to find but that, if he could secure a roommate, a space would be directly available. While departing, Bart reminded him of Monday night dinners. The man said he would be there (though his tone and manner immediately suggested that promise had been made before). The downtown corridor of West Walnut Hills begins two blocks east of the Kroger. The condition of the buildings looked no better than that of the ones we had already passed, but the architecture was markedly different: older, larger, more aesthetically creative. Bart explained that, in the 1920s and 1930s, this part of Cincinnati was popular and prosperous. Eighty years later, it is vacant and decrepit. Bart pointed to a massive building across the street, occupying nearly the whole length of the block. It is owned by a local “rich” man who repeatedly turns down buyers but 275 American Ethnologist • Volume 38 Number 2 May 2011 refuses to make any improvements. It seems a gigantic brick monument to neglect. Just as Bart was telling me about it, a middle-aged black woman walked by and affirmed his description: “That man is crazy,” she said of the owner. She informed us that he was recently hospitalized and not expected to live much longer. Walking away with her head tilted back toward us, she added that he also hordes all manner of things, the back door is never locked, and we could help ourselves anytime. Everyone else does. We walked farther east, then south, then east, then back north. We stood on the sidewalk of the major east-traveling artery, and Bart pointed to a collection of buildings just over a not-too-distant hill: East Walnut Hills. Bart carefully distinguishes East from West when talking to local Cincinnatians, so as not to be confused for one of the “rich, white” populace over the hill. He turned his indexing finger to a closer object, a high-rise brick building across the street from where we stood. Formerly a nationally known hotel, it had been shut down for a lengthy period. A private company bought it several years ago, using Housing and Urban Development money, and turned it into Section Eight housing. Rent is income based, and the unemployed can get a room for $25 a month. Several WHF members live there. “When their life falls apart, we find a place for them there.” Bart described the everyday happenings in and around the building as “not all that violent but still terrible.” We resumed walking, now heading west. We passed an elementary school to the north. Bart identified it as another governmental blunder. The building is only a few years old. Its multimillion-dollar construction had occurred in lieu of increasing teacher pay, increasing faculty size, and improving pedagogical resources. “It’s the same crappy school, it just looks pretty now.” A few hundred feet more, we turned south to find a public library. Part of Andrew Carnegie’s national library program, the building is 150 years old and the entrance is framed by two ornate pillars. Inside was an armed, black police officer. The library is not very spacious, but it is connected to the city’s other public libraries and most book requests are filled within a day. In a reading room, Bart saw a black woman he knew who looked to be in her early thirties. She was reading books by Joyce Meyer and Suze Orman. Bart greeted her, assessed her small pile of books, and smiled, “Prayer and financial freedom, I need both of those.” She laughed appreciatively. Much as he does the CVS and the Kroger, Bart considers this library a “positive space” and a crucial sliver of infrastructural hope for West Walnut Hills. A quarter-mile southeast of the library is a diner, the only one in the neighborhood. We dined on a sandwich and salad bar for a combined $12. Bart is a regular there and was immediately recognized by the waitress—a slightly overweight, beaming black woman in her early twenties. Most 276 of the customers are not local residents, but all of the staff is. Our lunch conversation heaped more details and stories onto an already dizzying afternoon. We traversed local employment rates; the neighborhood’s heavy reliance on government aid; family and gender dynamics; sexual and familial expectations for adolescent females; the cyclical nature of urban poverty; and the psychological trauma of love, need, neglect, and fear. As lunch neared its end, Bart spoke explicitly about the idea of place. He knows people who really care about locations, about architecture, green spaces, community development, and so on. He is not one of them. Place is “a means to an end of building relationships.” But he believes those relationships will never happen without a knowledge of place. It contextualizes those he wants to reach. “They could bulldoze the whole thing and it wouldn’t really make that much difference to me, but it would matter a lot for the people I work with on a daily basis.” He concluded, “If you are going to care for people, you have to care about what they do.” We left the diner and headed west, back toward the office. Across the busy street, Bart received a wave and a friendly shout from a middle-aged black man walking the opposite way. As we walked, Bart returned to themes of place and stability. He recalled how, when he and his family first moved to Walnut Hills, they lived in a bare apartment, cooking off a hot plate and sleeping on the floor. The forced intimacy had taken a toll on their relationships, and he realized that “if you’ve always had stable housing you can never understand this.” We turned the final corner, onto the street where my car was parked outside of Bart’s office. As we neared the office, he greeted three people sitting outside a dilapidated house in rickety lawn chairs. An older black man introduced Bart to a black man and woman in their twenties, both drinking 24-ounce cans of Budweiser. After some small talk, we continued to his office’s front stoop. We rested for a few minutes, trading reflections on the barrage of images and stories he had presented, and then shook hands good-bye. ∗∗∗ These two ethnographic accounts of neighborhood tours illustrate the sense of place cultivated by urban missional evangelicals. Their sense of place has three primary characteristics. First, it comprises a diverse body of knowledge that stresses intricate, nuanced details. Kevin and Bart nurture a competency in a wide range of affairs: racial privilege and disenfranchisement, local government decisions, high school sports, restaurants and other businesses, performances of social class, drugs, violence, literacy, automobiles, residential patterns, housing markets, employment, long-time residents, retail and commercial activity, Purity, danger, and redemption building history, infrastructure, masculine and feminine expectations, and federal governmental aid. Second, their sense of place features an integrated, multifaceted consciousness. Kevin and Bart demonstrate an attentiveness to multiple histories, social dynamics, and social semiotics, including race, class, economics, education, geography, gender, language, residency, criminality, and stability. In this way, place and religious subjectivity intersect, creating a lived experience of space that does not distinguish between enacting one’s faith, supporting community improvement efforts, awareness of social problems, and local memory. The unique familiarities Kevin and Bart foster of Middletown and West Walnut Hills recall the “intense particularity” that Feld and Basso (1996:11) consider so crucial for placed experience. Finally, and what most clearly distinguishes an evangelical attachment to locale, this intricately defined sense of place is mediate. It is, as Bart pithily stated, “a means to an end.” Sense of place is not the goal, nor does it define the conditions of the game; it is a tool for reaching people. Evangelism always takes precedence. Indeed, for urban missional evangelicals, place is only significantly useful when it helps strengthen religious community. The sense of place evident here is not, as Basso describes it, a form of cultural context that the ethnographer strains to piece together. The urban missional evangelical sense of place is not “vaguely realized most of the time, and rarely brought forth for conscious scrutiny” (Basso 1996:83). Rather, it is a strategic, and strategically made, lived reality in the ongoing attempt to, as Kevin would have it, “show them why Christ is more,” or as Bart would have it, “be in relationship with people.” This sense of place—intricately detailed, socially conscious, and mediate—is epiphenomenal. It occurs through the everyday performance of a missional identity. The most remarkable thing about Kevin and Bart is that there is absolutely nothing anomalous about their sense of place. The desire to be missional has spread widely through Emerging Church networks and institutions. Ministries in places like Middletown and West Walnut Hills are regularly featured in periodical articles, blogs, sermons, conference plenaries, and podcasts. Acts 29, in particular, is an institution that creates and circulates a large body of resources specifically devoted to explaining what it means to be missional and why the particularities of place matter. Ethnographic portraits of Kevin and Bart reveal a form of socially engaged evangelicalism that is distinct from the kind of suburban evangelical activism described by Elisha (2008b). Recalling the words of one of his ethnographic subjects, Emerging evangelicals work to not “just drop in, drop off, and leave the community behind” (Elisha 2008b:161). To conclude, I set the everyday religious subjectivity produced by being missional within the larger ideological contexts that frame this model of social engagement. • American Ethnologist Purity, danger, and redemption I have sketched the contours of how urban missional evangelicals dwell in place. Their emplacement in an urban setting occurs, in part, as a desire to separate from its symbolic opposite: suburbia and the conservative evangelical connotations associated with that locale. But there is more to the story than this. The missional-inspired sense of place is also a critique of modernity. Anthropologists working on the comparative study of Christianities, capitalizing on Max Weber’s classic analysis of Calvinism and capitalism, have devoted a good deal of attention to the relations between Christianity and the cultural conditions of modernity. Joel Robbins (2001) and Webb Keane (2002) have influentially argued that Western Protestant missionaries are bearers of distinctly modern conceptions: for example, a language ideology morally grounded in sincerity, which indexes Michel Foucault’s (1981) reading of modernity as an era obsessed by a will to truth. Susan Harding (2000) takes a similar stance in her ethnography of U.S. fundamentalists. Even though these conservative Protestants are frequently accused of being, and frequently claim to be, antimodern, they make ample use of modernist discourses, institutional configurations, and technologies. Elisha (2008a), in his reading of suburban evangelical activism, sees a clash between their forms of organizing and modern, liberalizing notions of humanism. Bialecki et al. correctly conclude that Christian communities manage a complex negotiation: “Christianity can serve at once as a vector for modernity and as counter-narrative to modernity” (2008:1151). Emerging evangelicals, at least in their desire and strivings to be missional, hope for the latter. In doing so, they reveal some discontents as late modern subjects. Given the various social science narratives about modernity, which is best suited to the case of urban missional evangelicals? I offer an unlikely candidate, one that has yet to be explicitly incorporated into the comparative study of Christianities: Mary Douglas’s Purity and Danger (1966). I say that Douglas is unlikely because her analysis focuses on “primitive” and “archaic” religions. But, on a closer look, she is clear in her assumption that there is “no special distinction between primitives and moderns: we are all subject to the same rules” (Douglas 1966:53). What rules? Douglas argues for the existence of an organizing binary: purity–danger. Purity subsumes order, control, sanctity, and holiness; and danger connects dirt, disorder, and pollution. “As we know it,” Douglas says, “dirt is essentially disorder” (1966:12), and “God’s work through the blessing is essentially to create order” (1966:63). Reading purity–danger as a narrative of modernity is affirmed by Ralph Cintron’s ethnography Angels’ Town: Chero Ways, Gang Life, and Rhetorics of the Everyday (1997). Cintron identifies discourses of measurement as a domineering 277 American Ethnologist • Volume 38 Number 2 May 2011 condition of modernity. Measurement values directly echo Douglas’s analysis of purity: coherence, control, completeness, precision, balanced form, order, and of course, cleanliness. Being modern is about embracing that which is clean and orderly and purifying that which is dirty and disorderly. I play on this general impulse that Douglas assigns to moderns and ask how it aligns with the Emerging evangelical critique of conservative Evangelicalism. In thinking through the effort to embrace what is pure and purify what is dangerous, Douglas writes, “Though we seek to create order, we do not simply condemn disorder” (1966:114), and “we must, therefore, ask how dirt, which is normally destructive, sometimes becomes creative” (1966:188). Disorder is not simply rejected or quarantined; it marks desired change and prompts creative action. How, then, do the dynamics of purity and danger figure into Emerging evangelical strivings to be missional? The answer begins with suburbia. Since its Levittown beginnings in the late 1940s, the land of ticky-tacky has held a special spot in the U.S. imagination. Consider the venerable presence of the suburb as a contextualizing symbol in U.S. popular culture. What would the TV series Leave It to Beaver (1957–63) or The Donna Reed Show (1958– 66) be without their iconically utopic setting? Less happily, Richard Yates’s novel Revolutionary Road (1961), ostensibly about the difficulties of modern marriage, entertains the psychological torment of living in the land of the setoff. Malvina Reynolds’s folky tune “Little Boxes” (1962) humorously invited a negative view of how suburban houses “all look just the same.” The kind of dark comedy hinted at by Reynolds is fully exploited in the film The Burbs (Dante 1989), in which Ray (played by Tom Hanks) wrestles to maintain the sanity that defeated Yates’s protagonists. Suburbia serves as the structural opposite of all that is interesting, exciting, and dangerous in the final scene of Goodfellas (Scorsese 1990). The film’s main character stands in his bathrobe, finally safe and unambiguously bored on his front porch, after having stood under threat of immediate death from New York “wiseguys.” Themes of psychological unrest and broken families continue in the film American Beauty (Mendes 1999), and a more whimsical version of suburban struggles plays on a loop in TV’s Desperate Housewives (2004–present). The Showtime series Weeds (2005– present) delights in hyperbole, using an underground marijuana ring headed by a seemingly together soccer mom, to unmask suburbia’s facade of contentment. To ensure the message is not mistaken, the opening credits roll while “Little Boxes” creaks in the background. As a cultural symbol, the suburb is both purity attained and danger thinly veiled by the false claim of attainment. With pop culture’s double-voiced interpretation fixed in the background, conservative Christianity’s relationship with suburbia began to solidify in the late 1960s. Eileen 278 Luhr argues forcefully for the close coupling of evangelical suburbanization, youth culture, consumerism, and conservative politics: “The suburban home came to be viewed both as the sentimental repository of established [family] values and economic success” (2009:6). This occurred alongside two demographic trends: a massive movement of Americans from the cities to the suburbs (half of the U.S. population called the land of cul-de-sacs home by 1990 [Luhr 2009:8]) and the racialization of that relocation, what Luhr and others call “white flight” (2009:9). The most notable (and visible) evangelical response to urban emigration was an organizational invention: the megachurch. The formula seems simple enough. Find an area of new residential growth and economic boom, provide a conveniently situated, ample building, and watch while people flood in. Roughly sketched, this is what happened. Megachurches proliferated on the U.S. religious landscape throughout the late 20th century, and suburbia became a promised land in the conservative evangelical imagination. These two late modern developments, the suburbanization of conservative evangelicalism and the rise of the megachurch, have become primary targets for the Emerging evangelical critique. Kevin, Bart, and my other 88 consultants articulate no shortage of objections to these trends, shifting attention from the suburb as purity attained to the suburb as danger thinly veiled. For them, the suburban megachurch is • • • • • • Tenuous. The money and resources needed to keep these institutions running is massive. They require constant maintenance and financial support from congregants. Any large decline in membership threatens a crumbling effect. Exaggerated. The hugeness that defines megachurches falls prey to the same logic that produces Wal-Marts and civilian Hummers. Size matters, and bigger is not better. Commodified. Megachurches are yet another iteration of conspicuous consumption. They fail to distance themselves from a pervasive social ill: the never-ending impulse to brand, package, mass produce, and generally plot everything in terms of buying and selling. Isolating. Intentionally set off in suburbia, adherents are removed from the conditions of urban need. This removal is tantamount to racial and class confinement and a gradual disappearance of nonmainstream experiences from everyday consciousness. Inauthentic. The size and easy anonymity of the megachurch is at odds with a hallmark of evangelicalism: the desire to cultivate spiritual intimacy (Bielo 2009b:73–92). Being deeply committed to a shared religious community becomes merely another option. Spectacle. The epicenter of megachurch life, the weekly worship event, is a mass-produced production for the masses. It is a show, in the worst sense of the word. It Purity, danger, and redemption plays into the U.S. preoccupation with the cult of fame by turning singers, musicians, and pastors into local celebrities. The Emerging evangelical attraction to being missional is closely coupled to this critique of the suburban megachurch. The desire of missionals to separate from conservative evangelicalism prompts an escape from suburbia. Unlike Elisha’s (2008b) evangelicals, who missionize the inner city via intermittent outreach, urban missional evangelicals seek to self-consciously dwell in their evangelizing contexts. One consequence of this is that the cultural logic guiding evangelical practice is altered. The two main categories identified by Elisha, compassion and accountability, do not disappear, but they are displaced by the ideal of being missional. This does not mean the kind of psychological–emotional struggle described by Elisha is absent among Emerging evangelicals, but it does mean that other cultural artifacts are more immediately visible. The example I have sketched here, cultivation of a sense of place, was most evident in my ethnographic fieldwork. This lived critique of the suburban megachurch is also, returning to Douglas (1966), a critique of modernity. Emerging evangelicals are discontent with the purification efforts of their conservative brethren. For them, the suburbanization of evangelicalism is a failed project of purity attainment, disorder mistook for order. By extension, long-distance urban evangelism that focuses strictly on the personal circumstances of the missionized subject is a misplaced attempt to purify what is out of order. In response, Emerging evangelicals seek to purify by attaching themselves to the disorder they hope to change. Redemption is the endgame. Kevin’s urban dwelling is directed toward showing Middletowners “why Christ is more” than their greatest hopes and fears. Bart is less convinced that everyday conditions can be dramatically changed. His urban dwelling is directed toward individualized relationships with people in an ongoing effort to “care” for them and restore “some dignity” to their lives. Emerging evangelicals “do not simply condemn disorder” (Douglas 1966:114); for them, the matter out of place, the dystopic conditions of urban decay, “becomes creative” (Douglas 1996:188). The sense of place they cultivate in the midst of seeking purity is always mediate, a vehicle for the change they desire. The irony that Douglas helps reveal in all of this is that Emerging evangelicals rely on a distinctly modern script to enact their critique of modern, suburban evangelicalism. To return to Bialecki et al., their escape from suburban disorder and embrace of urban disorder leaves Emerging evangelicals seeking a faith that is “at once [a] vector for modernity and [a] counter-narrative to modernity” (2008:1151). The case of urban missional evangelicals highlights an important dynamic that other anthropologists of Christianity will likely confront in their own ethnographic • American Ethnologist contexts. As Christian communities develop discontents with late-modern cultural conditions, they will respond by creating institutions and organizing modes of public engagement. What changes will result from such Christian encounters with perceived modern failures? What about place makes it a likely candidate to mediate these discontents? If the example of Emerging evangelicals like Kevin and Bart is instructive, these questions will have significant consequences for the future of late-modern places and the future of “place” in late modern Christianities. Notes Acknowledgments. I would like to thank two anonymous reviewers for their careful and insightful readings of this article and Donald Donham for his editorial guidance. Lydia Manning, M. Cameron Hay, John Cinnamon, and students in the spring 2010 sections of Anthropology 185: Cultural Diversity in the United States at Miami University also provided helpful feedback on a draft version. I enjoyed entertaining and insightful conversations about the place of suburbia in the U.S. imagination with Christopher Hensey and Brandon Ney. 1. I conducted research with Emerging evangelicals for three years, beginning in October 2007, including fieldwork in Michigan (Lansing, Grand Rapids), Arizona (Phoenix), and Ohio’s centralsouthwest corridor (Columbus, Dayton, Cincinnati, and their metropolitan statistical area outposts). This included three forms of data collection: (1) I collected primary texts produced by Emerging Church advocates and critics as well as secondary texts from secular and Christian media sources reporting on the movement; (2) I collected and transcribed podcasts, video posts, and rebroadcasts of interactions among and between Emerging Church advocates and critics; and (3) I conducted repeated formal and informal interviews with 90 individuals (representing 40 local communities and 11 denominations). This sample included lead and assistant pastors, lay scholars, church members, and Christian publishing agents. In 20 cases, I tracked individuals’ weblogs, a significant media form in the Emerging Church. In most cases, interviews led to observations of place and collective religious practice and the collecting of congregational materials. 2. A nagging problem for researchers and practitioners alike is calculating just how extensive the Emerging movement is. The only studied estimate I have seen claims 181 local churches of a definitively Emerging character in the United States (Flores 2005). Unfortunately, this count employs a far too restrictive criteria list, does not include Emerging constituents in established congregations, does not measure the number of practitioners in those 181 places, and focuses only on the local, congregational model of religious belonging. My own tally, a compilation of several Internet listings run by Emerging practitioners, produced roughly one thousand Emerging communities in the United States. 3. The 2000 census for West Walnut Hills recorded a population of 7,790, 85 percent of whom were black. 4. See Harding 2000 on conservative evangelicalism. See Bialecki 2009 for a progressive critique. References cited Acts 29 Network 2010 Doctrine: What We Believe and Why We Believe It. http://www.acts29network.org/about/doctrine/, accessed December 27. 279 American Ethnologist • Volume 38 Number 2 May 2011 Balmer, Randall 2006[1989] Mine Eyes Have Seen the Glory: A Journey into the Evangelical Subculture in America. New York: Oxford University Press. Basso, Keith H. 1996 Wisdom Sits in Places: Notes on a Western Apache Landscape. In Senses of Place. Steven Feld and Keith H. Basso, eds. Pp. 53–90. Santa Fe, NM: School of American Research Press. Bialecki, Jon 2009 Disjuncture, Continental Philosophy’s New “Political Paul,” and the Question of Progressive Christianity in a Southern California Third Wave Church. American Ethnologist 36(1):35–48. Bialecki, Jon, Naomi Haynes, and Joel Robbins 2008 The Anthropology of Christianity. Religion Compass 2(6):1139–1158. Bielo, James S. 2009a The “Emerging Church” in America: Notes on the Interaction of Christianities. Religion 39:219–232. 2009b Words upon the Word: An Ethnography of Evangelical Group Bible Study. New York: NYU Press. In press Emerging Evangelicals: Faith, Modernity, and the Desire for Authenticity. New York: NYU Press. Cintron, Ralph 1997 Angels’ Town: Chero Ways, Gang Life, and Rhetorics of the Everyday. Boston: Beacon Press. Dante, Joe, dir. 1989 The Burbs. 101 min. Imagine Entertainment. Beverly Hills, CA. Douglas, Mary 1966 Purity and Danger: An Analysis of Concepts of Pollution and Taboo. New York: Penguin Books. Elisha, Omri 2008a Faith beyond Belief: Evangelical Protestant Conceptions of Faith and the Resonance of Anti-Humanism. Social Analysis 52(1):56–78. 2008b Moral Ambitions of Grace: The Paradox of Compassion and Accountability in Evangelical Faith-Based Activism. Cultural Anthropology 23:154–189. Feld, Steven, and Keith H. Basso, eds. 1996 Senses of Place. Santa Fe, NM: School of American Research Press. Flores, Aaron O. 2005 An Explorationof the Emerging Church inthe United States: The Missiological Intent and Potential Implications for the Future. M.A. thesis, Department of Religion, Vanguard University. Foucault, Michel 1981 The Order of Discourse. In Untying the Text. Robert Young, ed. Pp. 51–77. Boston: Routledge and Kegan Paul. Goheen, Michael W. 2000 “As the Father Has Sent Me, I am Sending You”: J. E. Lesslie Newbigin’s Missionary Ecclessiology. Doctoral dissertation, University of Utrecht, the Netherlands. Hansen, Collin 2008 Young, Restless, Reformed: A Journalist’s Journey with the New Calvinists. Wheaton, IL: Crossways Books. 280 Harding, Susan 1987 Convicted by the Holy Spirit: The Rhetoric of Fundamental Baptist Conversion. American Ethnologist 14(1):167–181. 2000 The Book of Jerry Falwell: Fundamentalist Language and Politics. Princeton: Princeton University Press. Keane, Webb 2002 Sincerity, “Modernity,” and the Protestants. Cultural Anthropology 17(1):65–92. Low, Setha 2003 Behind the Gates: Life, Security, and the Pursuit of Happiness in Fortress America. New York: Routledge Press. Luhr, Eileen 2009 Witnessing Suburbia: Conservatives and Christian Youth Culture. Berkeley: University of California Press. Mendes, Sam, dir. 1999 American Beauty. 122 min. DreamWorks SKG. Glendale, CA. Newbigin, Lesslie 1983 The Other Side of 1984: Questions forthe Churches. Geneva: World Council of Churches. 1986 Foolishness to the Greeks: The Gospel and Western Culture. Grand Rapids, MI: William B. Eerdmans. Pals, Daniel L. 2006 Eight Theories of Religion. New York: Oxford University Press. Reynolds, Malvina 1962 Little Boxes. Berkeley, CA: Schroder Music. Robbins, Joel 2001 God Is Nothing But Talk: Modernity, Language, and Prayer in a Papua New Guinea Society. American Anthropologist 103(4):901–912. Sargeant, Kimon Howland 2000 Seeker Churches: Promoting Traditional Religion in a Nontraditional Way. New Brunswick, NJ: Rutgers University Press. Scorsese, Martin, dir. 1990 Goodfellas. 146 min. Warner Brothers. Burbank, CA. Webber, Robert 2002 The Younger Evangelicals: Facing the Challenges of the New World. Grand Rapids, MI: Baker Academic. Woolsey, Matt 2008 America’s Fastest-Dying Towns. Forbes Magazine. http:// www.forbes.com/2008/12/08/towns-ten-economy-forbeslifecx mw 1209dying.html, accessed December 27, 2010. Yates, Richard 1961 Revolutionary Road. Boston: Atlantic-Little, Brown. accepted November 11, 2010 final version submitted November 14, 2010 James S. Bielo Department of Anthropology Miami University 120 Upham Hall Oxford, OH 45056 bielojs@muohio.edu SPECIAL COLLECTION Islamic Sounds and the Politics of Listening Calling Everyone to Pray: Pluralism, Secularism, and the Adh?n in Hamtramck, Michigan Isaac A. Weiner, Ohio State University ABSTRACT This article critically interrogates the discourses of secularism and pluralism by analyzing their surprising effects in a 2003 dispute about the adh?n (Islamic call to prayer) in Hamtramck, Michigan. Hamtramck residents advanced different understandings of how secular governance should manage religious differences, but their arguments had unintended consequences that ran counter to their stated intents. In the end, I argue, Muslims were able to make themselves heard in Hamtramck, but only if they muted that which made their voices distinct. This article uses the Hamtramck dispute to analyze the particular conditions governing Islamic entry into the American public sphere. [Keywords: Hamtramck, Muslims, call to prayer, secularism, pluralism, sound, public] Introduction Caroline Zaworski was upset. “Muslims are allowed to pray in their mosque,” this 81-year-old, Polish-Catholic, lifetime resident of Hamtramck, Michigan declared at a contentious city council meeting in April 2004. “They are allowed to pray in their mosque, they can have their [call to prayer] in their mosque…that’s their right. But why is the loudspeaker so important? A Anthropological Quarterly, Vol. 87, No. 4, p. 1049-1078, ISSN 0003-5491. © 2014 by the Institute for Ethnographic Research (IFER) a part of the George Washington University. All rights reserved. 1049 Calling Everyone to Pray: Pluralism, Secularism, and the Adh?n in Hamtramck, Michigan holy prayer is a holy prayer. God hears it whether it’s on a loudspeaker, whether it’s in your heart, whether it’s in a mosque. Why agitate? Why bring all these dif?culties?”1 When the al-Isl?h Islamic Center’s leaders petitioned Hamtramck’s city council in January 2004 for permission to broadcast the adh?n, or call to prayer, they did not envision the ensuing “dif?culties” to which this neighbor referred. For six months, controversy raged in Hamtramck, attracting national attention as residents debated a proposed amendment that would exempt the adh?n from the local noise ordinance. The call to prayer functioned as a ?ashpoint in disputes about the integration of Muslims into this historically Polish-Catholic dominated community. No one openly contested Muslims’ right to worship in their mosques, but neighbors resisted and regarded as inappropriate this public pronouncement of Islamic presence that audibly intruded upon public space. Despite constitutional guarantees of free exercise, many suggested that there was a proper time, place, and decibel level for religious practice. Scholars have paid a great deal of attention recently to religion’s resurgent public presence (Meyer and Moors 2006; Butler et al. 2011; Calhoun, Juergensmeyer, and Van Antwerpen 2011). This literature has tended to focus on whether religious arguments have a legitimate place in democratic politics and legislative decision making. However, public debates have centered as much on different ways of using body and space as on different modes of public reasoning (Göle 2010b).2 Recent efforts to block Muslim communities from constructing new mosques or to ban Muslim women from wearing headscarves have demonstrated how particular places and practices have emerged as potent symbols in broader debates about secularism, pluralism, and the accommodation of religious differences (Bowen 2007, Scott 2007). These examples also indicate how debates about Islamic public presence have focused primarily on visual displays. Relatively little attention has been paid to the auditory modes through which Muslims have practiced their faith or to how sound has ?gured into broader efforts to demarcate religion’s proper place. Several scholars have noted music’s potential to bridge boundaries and mitigate religious, racial, and ethnic differences (Kun 2005, Kapchan 2008), but such boundary crossing can also seem threatening to unwilling listeners and unintended audiences. As in Hamtramck, sound has often marked the limit of what neighbors have been willing to tolerate. The burgeoning literature on sound studies has by now established that noise con?icts are rarely just about volume, for only certain sounds tend 1050 ISAAC A. WEINER to attract attention and elicit complaint. Noise has ?gured prominently in broader efforts to assert control over urban spaces, to delimit proper forms of behavior and expression, and to regulate who and what can be heard in public (Bailey 1998, Thompson 2002, Sterne 2005, Bijsterveld 2008). In fact, there is a long history in the US and elsewhere of dominant groups using noise ordinances to marginalize others. Sound studies scholarship has called particular attention to noise as an index of class and racial difference (Smith 2001, 2006). John Picker (1999), for example, has shown how Victorian professionals complained most stridently about the sounds of the urban poor. In the US, noise complaints have proven similarly useful for muting religious dissent. Groups such as the Salvation Army, the Jehovah’s Witnesses, and Hare Krishnas (ISKCON), who brashly and unabashedly brought their religion to the streets, have all faced accusations that they were too loud. At different historical moments, each found its practices restrained by nuisance suits, noise ordinances, and other legal mechanisms (Weiner 2014). The Hamtramck adh?n dispute was certainly similar to these earlier controversies. It brought to the fore simmering ethnic tensions in the city and gave voice to latent anti-Islamic prejudices. Long-time residents sought to alienate more recent newcomers by keeping their religious practices safely contained, by preventing the unwanted sounds of religious difference from spilling over into the surrounding neighborhood. Yet this case also played out very differently from most. While legal mechanisms have generally been deployed to silence the adh?n, Hamtramck’s City Council went out of its way to accommodate it. Even more surprising, Hamtramck residents went to the polls and voted to af?rm the council’s action, thereby using the mechanisms of democratic politics to make space for religious difference. The adh?n and its broadcasters were not marginalized, but af?rmed. In this article, I consider why the Hamtramck case study played out in these unexpected ways, and I analyze what we can learn from it about the conditions that govern Islamic entry into the American public sphere. I do this by unpacking a fundamental paradox that lay at its heart. As they contested the adh?n, Hamtramck residents appealed to very different understandings of how religious differences should be managed in a secular society. But their arguments had unintended effects that often ran counter to their stated intents. On the one hand, the adh?n’s critics interpreted secularism as requiring that public spaces be kept free from 1051 Calling Everyone to Pray: Pluralism, Secularism, and the Adh?n in Hamtramck, Michigan particularistic displays of religious expression. Yet their efforts to mute Islam’s public presence had the unintended consequence of making religion more audible in Hamtramck. Their resistance to the adh?n brought religious differences to the fore, making them the subject of public debate, discussion, and exchange. At the same time, the adh?n’s advocates framed their support in terms of a popular discourse of pluralism that celebrated religious variety. They argued that secular governance could accommodate diverse styles of religious expression in the public realm. Yet their efforts also bore unintended consequences, as they served more to efface differences than to bridge them, effectively muting the adh?n’s call even as they ostensibly made space for it. In the end, I argue, Muslims were able to make themselves heard in Hamtramck, but only in carefully prescribed ways.3 This article joins other works that have interrogated the particular terms on which Muslims have been incorporated into American society (Shryock 2010, Hicks 2012). For these purposes, I am less interested in how Hamtramck Muslims intended the adh?n to be heard or in what the adh?n dispute might teach us about new forms of Muslim subjectivity. Instead, I want to concentrate on how other Hamtramck residents responded to the adh?n’s call and to how their varied responses were shaped by broader assumptions about religion’s place in American public life. The parties to this dispute appealed to alternative modes of secular governance, which offered competing strategies for managing religious differences. Yet each of their approaches set tightly circumscribed limits on how those differences could be publicly articulated and performed. Each created particular conditions of possibility for public religious expression and, in so doing, transformed the adh?n’s meaning and message. Broadcasting the Call to New Audiences The of?cial website for Hamtramck, Michigan invites visitors to experience “a touch of the world in America.” The website includes pages written in Arabic, Polish, Albanian, and Serbo-Croatian. Storefront signs throughout the 2.1 square mile city similarly attract customers in a variety of languages. Diners can choose from a number of ethnic cuisines, including Lebanese, Mexican, Polish, Indian, Bangladeshi, Thai, Greek, and Ukrainian. Walking around town, one gets the sense that the website’s boast is not far from the truth. Yet it was not always this way in 1052 ISAAC A. WEINER Hamtramck. The call to prayer dispute emerged, in part, from the remarkable demographic, religious, and economic shifts that the city has experienced over the last several decades, as an almost exclusively PolishAmerican community has struggled to rede?ne itself. Named after a French-Canadian soldier in the 1790s, Hamtramck was incorporated as a politically independent city in 1922. It is surrounded by the city of Detroit on all sides, a distinct urban environment, set apart and de?ned in opposition both to downtown Detroit and its more af?uent suburbs. The construction of the Dodge Main factory in the 1910s and 1920s attracted thousands of Polish-Catholic auto workers to the area, and Hamtramck quickly became “a sort of island of Polish culture within the Metropolitan Area of Detroit,” as one sociological study described it in 1955 (Wood 1955:21). By the mid-1930s, its working-class population had reached close to 60,000, of whom over 80 percent were ethnically Polish. While small in area, Hamtramck was home to three Roman Catholic parishes, one Polish National Catholic Church, and numerous Polish restaurants and civic organizations (Kowalski 2002). By the latter decades of the 20th century, the city had begun to change in dramatic ways. A disastrous urban renewal project during the 1960s combined with problems in the automobile industry during the 1970s devastated Hamtramck’s economy, the ?nal blow coming when the Chrysler Corporation decided to close the Dodge Main in 1980. Thousands of Polish-American residents had long since migrated to the suburbs, and they gradually came to be replaced by an in?ux of ethnically diverse newcomers. Yemeni immigrants settled in Hamtramck’s south end. Its Eastern European population diversi?ed rapidly, as Albanians, Bosnians, Croatians, and Ukrainians moved to the town in large numbers. Immigrants from Bangladesh and other South Asian countries soon arrived, as well. These newcomers transformed the city’s ethnic composition, and they also began to revitalize its economy. The 2000 Census recorded the ?rst increase in Hamtramck’s population since 1930, rising from a low of 18,372 in 1990 to 22,976.4 Poles remained the single largest ethnic group, but constituted only 23 percent of the population. Other studies found that 41 percent of residents had been born outside of the US and that public school students spoke close to 30 different languages at home. Hamtramck had grown startlingly diverse. These newcomers brought with them new religious beliefs and practices. At the same time as the Detroit Archdiocese—facing an aging 1053 Calling Everyone to Pray: Pluralism, Secularism, and the Adh?n in Hamtramck, Michigan Catholic population—was closing all of Hamtramck’s parochial schools, mosques began to appear in converted of?ce buildings and storefronts. In fact, a vast majority of the new immigrants were Muslim, especially those who had arrived from Yemen, Bangladesh, and Bosnia. Arab immigrants and African-Americans had been building Islam in Detroit and nearby Dearborn since at least the ?rst half of the 20th century, but Muslims were relatively new to Hamtramck (Abraham and Shryock 2000). Their presence invited this historically Polish-American community to reimagine its distinct civic identity.5 Into this shifting social landscape ventured Abdul Motlib, a Bangladeshi factory worker, who had relocated his family from New York to Hamtramck in the late 1990s. In 2001, Motlib converted a former chiropractor’s of?ce into the al-Isl?h Islamic Center, a small, unassuming Bengali mosque located one block from Hamtramck’s primary commercial thoroughfare. In January 2004, Motlib approached Hamtramck’s city council to request permission to broadcast the adh?n from a small speaker on al-Isl?h’s roof. “As part of the Islamic religion,” he wrote in his petition, “it is our duty to ‘call’ all Muslims to prayer ?ve times a day.” “Why we make call to prayer?” he similarly explained to me: “We tell them, now our congregation prays in the mosque. If anyone wants to join, come.” But he also told me that he wanted to broadcast the call, in part, because with its “cheap, small houses,” and neighbors in close proximity, Hamtramck reminded him of Bangladesh, where hearing the adh?n constituted a regular feature of daily life. Similar to what Thomas Tweed (1997) found in his work on Cuban-American diasporic religion, the sound of the adh?n transported Motlib across time and space, forging a connection between Islamic practice, place, and identity. The call to prayer would help Motlib’s community make space for themselves in Hamtramck while connecting them to the place they had left behind. This public sound might help to place them in their new home. This interpretation seems consistent with what anthropologists of Islam have found elsewhere (Lee 1999, Metcalf 1996). The call to prayer echoes as one of the most distinctive features of Muslim cities and towns, with its interruption of daily routines ?ve times a day. Since its institution by the Prophet Muhammad, the adh?n has relied on a specially trained human voice, always male. At the appointed time, as set by the cycles of the sun (before dawn, noon, late afternoon, after sunset, and evening), the muezzin, or person responsible for giving the call, traditionally ascends to a designated space in a mosque’s minaret and reminds Muslims 1054 ISAAC A. WEINER of their obligation to “put aside all mundane affairs and respond to the call physically and spiritually” (Lee 1999:87). It has become increasingly common for mosques to use electric loudspeakers for this purpose, as they compete with each other to be heard amidst the din of urban life. Anthropologist Charles Hirschkind evocatively describes the effect of hundreds of mosques broadcasting the adh?n throughout Cairo, engul?ng the city “in a sort of heavenly interference pattern created by the dense vocal overlaying. These soaring yet mournful, almost languid harmonic webs soften the visual and sonic tyrannies of the city, offering a temporary reprieve from its manic and machinic functioning” (2006:124).6 Through its ritual enactment and prescribed text proclaiming God as uniquely worthy of worship, the adh?n differentiates Muslim time and space, distinguishing sacred from profane and holy from mundane (Metcalf 1996). The sound of the adh?n invests Islamic space with meaning, regulates the rhythms of daily life, and orients Muslims in relation to God and to each other. Borrowing composer R. Murray Schafer’s terminology, the adh?n constitutes a “soundmark,” analogous to a landmark, which “refers to a community sound which is unique or possesses qualities which make it specially regarded or noticed by the people in that community” (1993:10). Some scholars have even de?ned an Islamic community as including those within acoustic range of the muezzin’s call, its boundaries de?ned aurally rather than visually (Schafer 1993:215, Lee 1999:87). Broadcasting the call to prayer in a religiously pluralistic context also makes Muslims audible to others, however, and this has invited controversy numerous times. Lately, it has even become common for local zoning boards in the US to insist that mosque developers agree never to broadcast the adh?n publicly before granting them the necessary permissions.7 There have been notable disputes about the call to prayer in Germany, France, England, and other European countries (Eade 1996, Ewing 2002). Muslims in Detroit and Dearborn went to court to defend their right to amplify the adh?n in the late 1970s and early 1980s.8 In spring 2008, some Harvard University students even complained after Islamic Awareness Week organizers publicly broadcast the prayer call on campus (MacFarquhar 2008, Wieseltier 2008). In part to avoid these kinds of con?icts, many American mosques have turned their loudspeakers inwards (Smith 1999). They have chosen to avoid attracting unwanted attention by incorporating the adh?n into communal prayer, thereby transforming its meaning and function. 1055 Calling Everyone to Pray: Pluralism, Secularism, and the Adh?n in Hamtramck, Michigan Of course, religious rituals always take on new meanings in new contexts with new audiences, and this is perhaps no more true than when mosques broadcast the call to prayer throughout religiously diverse neighborhoods. After all, it was not only Muslims who comprised Hamtramck’s acoustic community. Calling Muslims to pray in Hamtramck, Michigan, in 2004, also meant audibly announcing Islamic presence to a Polish– Catholic community that was only starting to come to terms with the city’s changing demographics. Hearing the call would shape how other Hamtramck residents made sense of and responded to their new Muslim neighbors. For these neighbors, engaging with religious difference would not mean contemplating Islam as an intellectual abstraction, but rather making sense of the new sounds which were entering their shared city streets. Speci?c public practices would mediate their contact with these religious others and would invite—or demand—some kind of response.9 I do not mean to suggest that Abdul Motlib had all of these considerations in mind when he decided to broadcast the adh?n, nor that his act should be interpreted strictly in terms of...

 

Option 1

Low Cost Option
Download this past answer in few clicks

16.89 USD

PURCHASE SOLUTION

Already member?


Option 2

Custom new solution created by our subject matter experts

GET A QUOTE

Related Questions